LSAT and Law School Admissions Forum

Get expert LSAT preparation and law school admissions advice from PowerScore Test Preparation.

 Administrator
PowerScore Staff
  • PowerScore Staff
  • Posts: 8917
  • Joined: Feb 02, 2011
|
#40796
Complete Question Explanation
(The complete setup for this game can be found here: lsat/viewtopic.php?t=4676)

The correct answer choice is (D)

The question stem stipulates that a WZ block exists. This can occur under either template, creating the following modified templates:

PT63_Game_#2_#9_diagram 1.png
Answer choice (A) is incorrect because, as shown in Template #2, L could dive first.

Answer choice (B) is incorrect because T could dive third under either template:

..... Template #1 hypothetical: O-P-T-W-Z-L.
..... Template #2 hypothetical: L-P-T-W-Z-O.

Answer choice (C) is incorrect because, under Template #1, Z could dive third under this hypothetical: T-W-Z-O-P-L.

Answer choice (D) is the correct answer because P cannot dive fourth in either template; in both instances, placing P fourth would create problems in placing the WZ block.

Answer choice (E) is incorrect because Z could dive fourth under either template:

Template #1 hypothetical: O-T-W-Z-P-L.
Template #2 hypothetical: L-T-W-Z-P-O.
You do not have the required permissions to view the files attached to this post.

Get the most out of your LSAT Prep Plus subscription.

Analyze and track your performance with our Testing and Analytics Package.